Carson has a smart phone data plan that costs $35 per month that includes 10 GB of data, but will charge an extra $15 per GB over the included amount. How much would Carson have to pay in a month where he used 3 GB over the limit? How much would Carson have to pay in a month where he used went over by x GB?

Answers

Answer 1

Answer:

3 GB over: $80 x GB over: $35+$15x

Step-by-step explanation:

First, 3 GB over is 3 times 15 plus 35, which equals 80. x GB over is x times 15 plus 35, hence the answer. Hope this helps!

Answer 2

For using 3GB of data more, she'll be charged $80.

a. Amount charged per month = $35

Amount charged for over-usage = $15 per gb.

Therefore, when she uses 3gb more, the amount charged will be:

= $35 + ($15 × 3)

= $35 + $45.

= $80

When she uses x gb more, the amount that she'll be charged will be:

= $35 + ($15 × x)

= $35 + 15x

= 35 + 15x

Read related link on:

https://brainly.com/question/23306305


Related Questions

Select all that apply.
The slope of the graph of a direct variation function is -3. Which of the following is true?

The equation is y = -3x.
The function is linear.
The equation of the function is y = -x .
The point (2, -6) is on the graph of the function.

Answers

a and b are correct

c is wrong

d can't be decided, because it can be true for one case, but not for others.

so only go for a and b

Answer:

B. The function is linear. (A and D COULD work)

Step-by-step explanation:

If the graph of a direct variation function has a slope of -3, then we know that the function is linear, and that m in the equation y = mx + b is -3. However, option A could be incorrect since we don't know if there is a y-intercept or not. C is wrong because of -3 being the slope, and D would only be true if A was also correct. So that leaves B. (If you know that there is more than one option, then A, B, and D are correct.)

A library has 160 books. 3/8 are story books, 1/4 are poetry books and the remainder are books. What fraction is comic books?

Answers

Answer:

1/8

Step-by-step explanation:

1/4*2/2=4/8

4/8+3/8=7/8

1-7/8=1/8

solve
[tex]100 {5}^{2} - 500 {5}^{6} [/tex]
Please help before 9:00 pm​

Answers

1005 to the power of 2 is 1010025 sorry if i'm wrong, and 5005 to the power of 6 is 1.5718985e+22. so 1.5718985e+22-1010025=-1.5718985e+22. wow my answer is so random ofc it's not correct lollllll i'm sorry i'm horrible at math and i barely understood this equation so i am super super super sry :(

On a piece of paper, graph y< - 1/2 x + 1

Answers

Answer:

Image below

Step-by-step explanation:

Make sure you draw it with a dotted line.

can someone help me pls

Answers

Answer: No solution

Step-by-step explanation:

First, rearrange the second function:

[tex]3x - y = 2 \\-y=-3x+2\\y=3x-2[/tex]

Because the two equations share the same slope of 3, they're parallel lines, that means they don't ever intercept. Therefore, there are no solutions.

What is the equation of the following line? Be sure to scroll down first to see
all answer options.
10
(-4,8)
(0,0)
. 10
10

Answers

The answer is y= -2x.

The equation of the line passing through points (0, 0) and (-4, 8) is y = -2x

What is an equation of line?

The equation of line is an algebraic form of representing the set of points, which together form a line in a coordinate system.

Given is a graph of a line passing through points (0, 0) and (-4, 8), we need to find the equation of the line,

We know that, the equation of a line passing through two points is given by,

y-y₁ = y₂-y₁ / x₂-x₁ (x-x₁)

Here, (x₁, y₁) and (x₂, y₂) are (0, 0) and (-4, 8)

So, the equation of the line =

y-0 = 8/(-4) (x-0)

y = -2x

Hence, the equation of the line passing through points (0, 0) and (-4, 8) is y = -2x

Learn more about equation of line, click;

https://brainly.com/question/21511618

#SPJ7

Urgent i need help!!…….

Answers

Answer:

Step-by-step explanation:

These are similar triangles. We know that because we know that all right triangles are similar. The height of the red one is 8 and the height of the blue one is 4; that means that the red one is twice the size of the blue one; likewise, the blue one is half the size of the red one. That means that ALL the measurements of these triangles exist in that ratio...even the base of the blue one. If the base of the red one is 3, and the red one is twice the size of the blue one, then the base of the blue one is 3/2 or 1.5. I can't see your choices because they are too small.

it should be 1.5……………. if it’s wrong sorry

what is the quotient

Answers

Answer:

The quotient is 4x+5  and  the remainder is -13

Step-by-step explanation:

Using synthetic division

-5        4      25   12

Bringing down the 4 and multiplying

-5        4      25   12

        ↓       -20 -25

    ----------------------

         4        5    -13

The quotient is 4x+5  and  the remainder is -13

Answer:

  4x   +5    [tex]\frac{-13}{x+5}[/tex]

Step-by-step explanation:

          .           4x   +5    [tex]\frac{-13}{x+5}[/tex]

x + 5 | 4x^2 + 25x + 12

        - 4x^2 + 20x

                         5x + 12

                         5x + 25

                               -13

Keith used the following steps to find the inverse of f, but he thinks he made a error
f(x) = 7x + 5

Answers

Answer:

Step-by-step explanation:

[tex]\Large \boldsymbol{} f(x) \ \ inverse \ \ function \ \ (f(x))^{-1} \\\\ y=7x+5 \\\\x=7y+5 \\\\ y=\dfrac{x-5}{7} \ \ or \ \ f(x)^{-1}= \dfrac{x-5}{7}[/tex]

What is the product?
(-2d^2+s)(5d^2-6s)

Answers

Answer:

Option 1: -10d⁴ + 17d²s - 6s²

General Formulas and Concepts:

Algebra I

Terms/CoefficientsExpand by FOIL

Step-by-step explanation:

Step 1: Define

Identify

(-2d² + s)(5d² - 6s)

Step 2: Expand

FOIL:                                                                                                                  -10d⁴ + 12d²s + 5d²s - 6s²Combine like terms:                                                                                         -10d⁴ + 17d²s - 6s²

Which number produces a rational number when added to 0.25? ​

Answers

The step of adding it to 0.25 (or 25/100, or 1/4) is indeed not relevant. 0.25 is rational.

A rational number added to a rational number gives a rational number. A rational added to an irrational number doesn't.

so effectively the task can be reduced to "wich one of these is a rational number?"

And that's only true for A. 2/9

All others can't be represented as a discrete/finite fraction / ratio.

The sum of two rational numbers is a rational number thus option (A) is correct.

What is an irrational number?

Any real number that cannot be written as the quotient of two integers, p/q, where p and q are both integers, is referred to as an irrational number.

For example, √2, and √3 are irrational numbers because they cannot be written as p/q where p and q both should be integers.

The sum of two rational numbers gives the rational number.

Since 2/9 is written in p/q where 2 and 9 both are integers thus it will be rational.

So, 2/9 + 0.25 = 2/9 + 1/4

(8 + 9)/36 = 17/36 here 17 and 36 both are integers thus it will be rational.

Hence "A rational number is the result of two rational numbers".

For more about the irrational number,

https://brainly.com/question/4031928

#SPJ2

Calculate.
12C4

Please help!

Answers

Answer:

495

Step-by-step explanation:

[tex] {12 \: }^{c} \: 4 = \frac{12!}{4!(12 - 4)!} \\ = \frac{12!}{4! \times 8!} \\ = \frac{12 \times 11 \times 10 \times 9 \times 8!}{4! \times 8!} [/tex]

canceling 8! from both numerator and denominator.

[tex] \frac{12 \times 11 \times 10 \times 9}{4 \times 3 \times 2 \times 1} \\ = 11 \times 9 \times 5\\ = 495[/tex]

Factorial of an integer, in mathematics, is the product of all positive integers less than or equal to the given positive integer.

Factorial of a number is denoted by placing an exclamation mark behind the number

[tex]\quad\quad\quad\quad \net\net \net\netu\internetu\y\menyaa \la\la\la\la\\\\\\\\\\\\\dddddddddd\\\FFFFFFFF\\\\\\\\\\\\\ \\\\\\\\\\\\\\\\\\\\\\\\\\\\\\\\\\\\\\\\\\\\\\\\\displaystyle\ \Large \boldsymbol{} _{12}C_4=\frac{12!}{4!(12-4)!} =\frac{12!}{4!\cdot8!} =\frac{12\cdot11\cdot10\cdot9\cdot8!\!\!\!\!\!\diagup}{4!\cdot8! \!\!\!\!\!\diagup} = \\\\\\\frac{12\!\!\!\!\!\diagup\cdot11\cdot10\cdot9}{24\!\!\!\!\!\diagup_{\ \ 2}}= \frac{11\cdot10\cdot9}{2} =\boxed{495}[/tex]

*Please Help!*

What is the volume of water, to the nearest tenth of a cubic metre, that would fill this spa tub?

First cylinder= 0.75m diameter, 0.80m height

Cylinder Underneath= 1.25m diameter, 0.70m height

Semi Sphere that holds both cylinders= 3m long

Answers

Answer:

The volume of water that will fill the spa tub is 5.9 cubic meters.

Step-by-step explanation:

Volume of water that would fill the spa tub = volume of semi sphere - (volume of the first cylinder + volume of the second cylinder)

i. volume of first cylinder = [tex]\pi[/tex][tex]r^{2}[/tex]h

where r is the radius and h is the height of the cylinder.

r = [tex]\frac{0.75}{2}[/tex] = [tex]\frac{3}{8}[/tex]

 = 0.375 m

h = 0.80 m

volume of the first cylinder = [tex]\frac{22}{7}[/tex] x [tex](\frac{3}{8} )^{2}[/tex] x 0.8

                                        = 0.3536 cubic meters

ii. volume of the cylinder underneath = [tex]\pi[/tex][tex]r^{2}[/tex]h

r = [tex]\frac{1.25}{2}[/tex] = [tex]\frac{5}{8}[/tex]

 = 0.625

h = 0.70 m

volume of the cylinder underneath = [tex]\frac{22}{7}[/tex] x [tex](\frac{5}{8}) ^{2}[/tex] x 0.7

                                                      = 0.8594 cubic meters

iii. volume of the semi sphere = [tex]\frac{2}{3}[/tex] [tex]\pi[/tex][tex]r^{3}[/tex]

where r is the radius = 1.5 m

volume of the semi sphere = [tex]\frac{2}{3}[/tex] x [tex]\frac{22}{7}[/tex] x [tex](1.5)^{3}[/tex]

                                             = 7.0714 cubic meters

Thus,

volume of the water to fill the spa tub = 7.0714 - (0.3536 + 0.8594)

                                                     = 5.8584

The volume of water that will fill the spa tub is 5.9 cubic meters.

Find the length of BC

Answers

Answer:

Hope this helps.. Have a good day

Full working out for these questions please.
First question is the first attachment and the second question is the second attachment.

Answers

Answer:

c,  

Step-by-step explanation:

Explain how to solve 5^(x-2)= 8 using the change of base formula

Answers

Answer:

x = 3.3

Step-by-step explanation:

A equation is given to us and we need to solve out for x. The given equation is ,

[tex]\sf\longrightarrow 5^{x -2}= 8 [/tex]

Take log on both sides with base as " 10" . We have ,

[tex]\sf\longrightarrow log_{10} 5^{x-2}= log_{10}\ 8[/tex]

Simplify using the property of log , [tex]\sf log a^m = m log a [/tex] , we have ,

[tex]\sf\longrightarrow ( x -2) log_{10} 5 = log_{10} 8 [/tex]

Simplify ,

[tex]\sf\longrightarrow ( x -2 ) log_{10}5 = log_{10} 2^3[/tex]

Again simplify using the property of log ,

[tex]\sf\longrightarrow (x-2) log 5 = 3 log 2[/tex]

We know that log 5 = 0.69 and log 2 = 0.301 , on substituting this , we have ,

[tex]\sf\longrightarrow ( x - 2 ) = \dfrac{ 3\times 0.301}{0.69}[/tex]

Simplify the RHS ,

[tex]\sf\longrightarrow x - 2 = 1.30 [/tex]

Add 2 both sides ,

[tex]\sf\longrightarrow \boxed{\blue{\sf x = 3.30}}[/tex]

Hence the Value of x is 3.30 .

Answer:

its actually 3.292 because we round to the nearest thousandth and thats not even the equation you use above

Step-by-step explanation:

For this equation we use the formula log a^m=m (log a) so the equation will be written as log 5 (5^x-2) = log 5 (8). You use the base, which is 5, and use log to base 5 on both sides of the equation. Then you take the exponent " x-2" and write( x-2) log 5 (5) = log 5(8). Since log a =1, you multiply that 1 by x-2, which keeps it x-2. Making the equation x-2 = log 5 (8). Next, we use the change of the base properties with the formula log b^y= log y/ log b. The equation will be written as x-2 = log 8/ log 5, since 5 is the base it stays in the bottom or basement. We then add +2 to both sides of x-2 and log 8/ log 5. To solve this equation, you can find out what log 8 and log 5 are and divide those and add +2 to solve. So log 8 = 0.903 and log 5 = 0.698970 and divide those to get 1.29190 +2 and you get the answer rounded as 3.292. 

v=u + 2at
Where v is the final velocity (in m/s), u is the initial velocity (in m/s), a is the
acceleration (in m/s?) and t is the time in seconds).
Find v when u is 35 m/s, a is 28 m/s2, and t is 58 seconds.

Answers

Answer:

3283m/s

Step-by-step explanation:

V=U+2at

V=35+2(28)(58)

V=35+3248

V=3283m/s

Gary's income and expenses in a month are 4,000 and 3,360 respectively.
a) What percentage of his income are expenses?
b) What percentage of his income is savings?
(SHOW YOUR WORK!)

Answers

Answer:

84%

Step-by-step explanation:

3360/4000=.84

.84=84%

a) Arrange the following numbers so that the sum of any two neighbouring numbers is a perfect square.
7 , 11 , 9 , 14 , 16 , 2 , 25
Ex. 3+6 = 3^2 and 6+10 = 4^2

b) Find a way to arrange all of the intervenes from 1 to 17 to a list with the property from part a)

Answers

I'll do part (a) to get you started.

One possible answer to part (a) is 25,11,14,2,7,9,16

There are likely other possible answers.

The explanation is below.

==============================================================

Let's say 7 is the anchor value and we want to see which values could be its next door neighbor.

It can't be 11 since 7+11 = 18 isn't a perfect square.But 9 works because 7+9 = 16 is a perfect square (4^2 = 16)14 doesn't work because 7+14 = 21 isn't a perfect square16 doesn't work either since 7+16 = 23 isn't a perfect square2 however does work since 7+2 = 9 is a perfect squareLastly, 25 won't work because 7+25 = 32 isn't a perfect square

To summarize this subsection, the anchor value 7 could have the neighbors 9 and 2.

So we could have 2,7,9 or 9,7,2 as a subsequence. We'll keep this in mind for later.

------------------------------------------------

Now we'll make 11 the anchor. We already checked 7 and it doesn't work.

9 doesn't work either because 11+9 = 20 isn't a perfect square

11+14 = 25 does work11+16 = 27 doesn't work11+2 = 13 doesn't work11+25 = 36 does work

Of that list, only 14 and 25 are possible neighbors of the anchor value 11.

So we could have the subsequence 14,11,25 or 25,11,14.

------------------------------------------------

If 9 is the anchor, then,

9+7 = 16 works like we found earlier (section 1)9+11 = 20 doesn't work9+14 = 23 doesn't work9+16 = 25 does work9+2 = 11 doesn't work9+25 = 34 doesn't work

The values 7 and 16 are possible neighbors of 9. We could have the subsequence 7,9,16 or 16,9,7

Let's go back to the subsequence 2,7,9 and tack 16 at the end to get 2,7,9,16

------------------------------------------------

If 14 is the anchor, then,

7 doesn't work because 14+7 = 2111 does work either because 14+11 = 259 doesn't work because 14+9 = 2316 doesn't work because 14+16 = 302 does work because 14+2 = 1625 doesn't work since 14+25 = 39

We could have the subsequence 11,14,2 or 2,14,11

Let's go with the first option and stick "11,14,2" in front of "2,7,9,16" to end up with the larger subsequence 11,14,2,7,9,16

We can then stick 25 at the front because 25+11 = 36 is a perfect square.

------------------------------------------------

So one possible sequence of values is 25,11,14,2,7,9,16

Here's the verification

25+11 = 36 is a perfect square (6^2 = 36)11+14 = 25 is a perfect square (5^2 = 25)14+2 = 16 is a perfect square (4^2 = 16)2+7 = 9 is a perfect square (3^2 = 9)7+9 = 16 is a perfect square (4^2 = 16)9+16 = 25 is a perfect square (5^2 = 25)

Each pair of adjacent terms add up to a perfect square, so the answer is confirmed.

There are probably other solutions as well.

Side note: The video math channel "Numberphile" has a video discussing this topic in which you might be interested in. Search out "The square sum problem" with quotes (the presenter/teacher in the video is Matt Parker).

Answer:

25, 11, 14, 2, 7, 9, 16.

Step-by-step explanation:

25 + 11 = 6^2

11 + 14 = 5^2

14 + 2 = 4^2

2 + 7 = 3^2

7 + 9 = 4^2

9 + 16 = 5^2

Which digit in 123.456 has highest value?
Digit
6

5

4
1

Answers

Which digit in 123.456 has highest value?

answer = 1....

Answer:

1

Step-by-step explanation:

The 1 is in the hundredth's place and it has the highest value

find the length of a side and perimeter of square those area is 256 square​

Answers

you have to add the square root then do the division 54 is the answer

Answer:

x2 = 256; 16 m

Step-by-step explanation:

Let x=​  the length of the side ofthe square. So an equation tomodel this problem is x2=256.x2xx===256256⎯⎯⎯⎯⎯⎯√± 16The side of a square with an areaof 256 m2 is 16 m.

Are all numbers natural numbers

Answers

The whole numbers are the numbers 0, 1, 2, 3, 4, and so on the natural numbers and zero. Negative numbers are not considered "whole numbers." All natural numbers are whole numbers, but not all whole numbers are natural numbers since zero is a whole number but not a natural number.

Please solve this problem 9p-10=6+p?

Answers

Answer:

p=2

Step-by-step explanation:

9p-10=6+p

Subtract p from each side

9p-p-10=6+p-p

8p -10 =6

Add 10 to each side

8p -10+10 =6+10

8p =16

Divide each side by 8

8p/8 = 16/8

p =2

Answer:

p = 2

Step-by-step explanation:

1. 9p - 10 = 6 + p

2. 8p - 10 = 6

3. 8p = 16

4. p = 2

Is this true or false

Answers

Answer:

I say it's TRUE

Step-by-step explanation:

I hope this help you

I assume that it’s true

Write (4√81)^6 in exponential form.

Answers

Answer:

Step-by-step explanation:

[tex](4\sqrt{81})^{6} = (4*\sqrt{9*9} )^{6}\\\\=(4*9)^{6}\\\\=(2^{2}*3^{2})^{6}\\\\=[(2*3)^{2}]^{6}\\\\=(6)^{2*6}\\\\=6^{12}[/tex]

Exponent Laws:

[tex](a^{m}*b^{m}) = (a*b)^{m}\\\\(a^{m})^{n}=a^{m*n}[/tex]

[tex]\huge\textsf{Hey there!}[/tex]

[tex]\mathsf{(4\times\sqrt{81})^6}\\\\\mathsf{\sqrt{81} = \bf 9}\textsf{ because half of \underline{81} is \underline{9} or 9 multiplied by itself gives you 81}\\\\\mathsf{= (4\times9)^6}\\\\\mathsf{4\times9 = \bf 36}\\\\\mathsf{36^6}\\\mathsf{= 36\times36\times36\times36\times36\times36}\\\mathsf{= 1,296\times1,296\times1,296}\\\mathsf{= 1,679,616\times1,296}\\\mathsf{= \bf 2,176,782,336\ also\ known\ as \ 6^{12}}\\\\\\\boxed{\boxed{\large\textsf{Answer: \bf 6}^{\bf 12}}}\huge\checkmark[/tex]

[tex]\large\textsf{Good luck on your assignment and enjoy your day!}[/tex]

~[tex]\frak{Amphitrite1040:)}[/tex]

A cash register at a store contains $277 bills. There are six more $5 bills than $10 bills. The number of $1bills is two more than 24 times the number of $10 bills. How many bills of each kind are there

Answers

Answer: See explanation

Step-by-step explanation:

Your question isn't well written, the store contains $227 and not $277.

let c be the number of $1 bills

let f be the number of $5 bills

let t be the number of $10 bills.

Therefore, the equation will be:

1c + 5f + 10t = 227

Since there are six more $5 bills than $10 bills, therefore,

f = t+6

Also, the number of $1 bills is two more than 24 times the number of $10 bills and this will be:

c = 24t + 2

Now, we substitute the equation for f and c into the main equation and this will be:

1c + 5f + 10t = 277

1(24t + 2) + 5(t + 6) + 10t = 227

24t + 2 + 5t + 30 + 10t = 227

39t + 32 = 227

39t = 227 - 32

39t = 195

t = 195/39

t = 5

Therefore, there are 5 $10 bills

Since c = 24t + 2

c = 24(5) + 2

c = 120 + 2

c = 122

Therefore, there are 122 $1 bills

Since f = t+6

f = 5+6

f = 11

Therefore, there are 11 $5 bills.

Can someone help me with this math homework please!

Answers

9514 1404 393

Answer:

7/3-3y = 7/3x - 3

Step-by-step explanation:

1) The slope is given by the formula ...

  m = (y2 -y1)/(x2 -x1)

  m = (4 -(-3))/(3 -0) = 7/3

__

2) The y-intercept is the point where x=0:

  (x, y) = (0, -3) . . . . . y = -3

__

3) Then the slope-intercept equation is ...

  y = mx + b . . . . . . . line with slope m and y-intercept b

  y = 7/3x -3

Find the center and radius of the circle whose equation is x 2 + y 2 - 6x - 2y + 4 = 0

Answers

Answer:

[tex]{ \tt{ {x}^{2} + {y}^{2} - 6x - 2y + 4 = 0 }} [/tex]

Center:

[tex]{ \tt{(3, \: 1)}}[/tex]

Radius:

[tex]{ \tt{radius = \sqrt{ {g}^{2} + {f}^{2} - c } }} \\ = \sqrt{36 + 4 - 4} \\ = 6 \: units[/tex]

An American sausage is 4/3 inches long. How many pieces of sausage can be cut from the 4/3 inch piece of sausage if each piece is two-thirds of an inch?

Answers

Answer:

2 Pieces can be cut

Step-by-step explanation:

We know that our sausage is 4/3 inches and we want to know how many 2/3 pieces can be cut. we start by examining our fractions.

4/3 & 2/3, since they have the same denominator we can simply divide the total length by how big each cut should be!

4/3 ÷ 2/3 = 2

expand this question (x+5)(x-3)

Answers

X•x + x•-3 + 5•x +5•-3
X^2+-3x+5x+-15
X^2+2x-15
Other Questions
100 POINTS!!!!!! Help FAST!!!!!!!!!!!!!!The picture below shows the bone structures of human, cow, and horse.Picture shows similar bone structures of human, cow, and horse.Based on the diagram, which of these statements is correct about humans, cows, and horses?They have developed from the same organism.They have completed their evolutionary process.Humans have evolved from several ancestors.Horses have evolved from cows and humans. Rice is my favourite dish. Which word is an uncountable noun? * ) dish O O O O favourite rice Carter Industries has two divisions: the West Division and the East Division. Information relating to the divisions for the year just ended is as follows: West East Units produced and sold 33,000 43,000 Selling price per unit $ 8 $ 15 Variable costs per unit 4 5 Direct fixed cost 51,000 113,000 Common fixed cost 43,000 43,000 Common fixed expenses have been allocated equally to each of the two divisions. Carter's segment margin for the West Division is: You are required to copy in your attendees' emails from outside of Outlook when creating a meetingO TrueO False Refrigerant-134a enters an adiabatic compressor at -30oC as a saturated vapor at a rate of 0.45 m3 /min and leaves at 900 kPa and 55oC. Determine (a) the power input to the compressor, (b) the isentropic efficiency of the compressor, and (c) the rate of exergy destruction and the second-law efficiency of the compressor. Take T0 does anyone know how to do this help please What is the system of inequalities associated with the following graph What do these lines tell you about the attitude men on Earth had toward any men on Mars?It is curious to recall some of the mental habits of those departed days. At most terrestrial men fancied there might be other men upon Mars, perhaps inferior to themselves and ready to welcome a missionary enterprise. (4 points)Select one:a. If they existed, they could teach humans things.b. If they existed, they would leave humans alone.c. If they existed, they would be interested in humans.d. If they existed, they would need help from humans. Spanish help I have no idea why is wrong SOMEONE HELP ME PLEASE Solve the equation for all values of xx.3x(x3)(x 2 +16)=0 One of the social "cons" of the Industrial Revolution was the large monetary gap that was suddenly created within the classes of people. Why was this gap a problem for society? if all possible diagonals formed are 740 then find the number of sides 2. Given f(x) = 4x -2 and g(x) = x2 + 1, evaluate:a) f(5)b) g(-3) c) g(x - 2)d) f(g(x)) The Fifteenth Amendment to the United States Constitution a.ended the institution of slavery and indentured servitude throughout the United States. b.prohibited federal and state governments from denying any citizen the vote because of race. c.guaranteed women the right to vote in federal elections but did not allow them to vote in state elections. d.made the income tax constitutional for the top 30 percent of wealthy white Americans but not black southerners. which choice is equivalent to the expression below? square root of 50 - square root of 2 Qu factores geogrficos y culturales justificaron la expansin imperial europea sobre Asia y frica? Which highlighted word is an adjective?A lake with clean water is a wonder to behold.A. cleanB. wonderC. behold Triangle A B C is a right triangle. Find the missing side length. a = 12, b = ? , c = 15Please answer becuase Im having a really hard time lol Thanks! Help me determine Rise/run